If a sports team is up $2-1$ in a best of $7$ series, what's the probability that they will win the series?...












0












$begingroup$



This question already has an answer here:




  • If a sports team is down by 1-2 in a best of 7 series, what is their chance of winning (if both teams have a 1/2 chance of winning every game)?

    2 answers




Assuming that each team has an equal $frac{1}{2}$ probability of winning each game, and the probability of winning each game is independent.



I solved this using a tree diagram, and got the answer of $frac{11}{16}$, but I want to figure out a way of solving this without relying on a tree.



I tried diving this up to cases where the series ends in 2 games, 3 games and 4 games, but am having difficulty.



Any help would be greatly appreciated!










share|cite|improve this question











$endgroup$



marked as duplicate by David G. Stork, lulu, Key Flex, N. F. Taussig combinatorics
Users with the  combinatorics badge can single-handedly close combinatorics questions as duplicates and reopen them as needed.

StackExchange.ready(function() {
if (StackExchange.options.isMobile) return;

$('.dupe-hammer-message-hover:not(.hover-bound)').each(function() {
var $hover = $(this).addClass('hover-bound'),
$msg = $hover.siblings('.dupe-hammer-message');

$hover.hover(
function() {
$hover.showInfoMessage('', {
messageElement: $msg.clone().show(),
transient: false,
position: { my: 'bottom left', at: 'top center', offsetTop: -7 },
dismissable: false,
relativeToBody: true
});
},
function() {
StackExchange.helpers.removeMessages();
}
);
});
});
Jan 16 at 19:59


This question has been asked before and already has an answer. If those answers do not fully address your question, please ask a new question.


















  • $begingroup$
    An alternative to dividing this up into cases where the series ends in $2$ games, $3$ games and $4$ games, is to play all $4$ remaining games (some unnecessarily): this team need to win $2$, $3$ or $4$ of them to win the series
    $endgroup$
    – Henry
    Jan 16 at 18:23










  • $begingroup$
    I checked that thread out, but the solution given uses a tree or assuming all of the remaining 4 games get played out. I'm trying to use cases where not all 4 games necessarily get played out.
    $endgroup$
    – Panaso
    Jan 16 at 18:33
















0












$begingroup$



This question already has an answer here:




  • If a sports team is down by 1-2 in a best of 7 series, what is their chance of winning (if both teams have a 1/2 chance of winning every game)?

    2 answers




Assuming that each team has an equal $frac{1}{2}$ probability of winning each game, and the probability of winning each game is independent.



I solved this using a tree diagram, and got the answer of $frac{11}{16}$, but I want to figure out a way of solving this without relying on a tree.



I tried diving this up to cases where the series ends in 2 games, 3 games and 4 games, but am having difficulty.



Any help would be greatly appreciated!










share|cite|improve this question











$endgroup$



marked as duplicate by David G. Stork, lulu, Key Flex, N. F. Taussig combinatorics
Users with the  combinatorics badge can single-handedly close combinatorics questions as duplicates and reopen them as needed.

StackExchange.ready(function() {
if (StackExchange.options.isMobile) return;

$('.dupe-hammer-message-hover:not(.hover-bound)').each(function() {
var $hover = $(this).addClass('hover-bound'),
$msg = $hover.siblings('.dupe-hammer-message');

$hover.hover(
function() {
$hover.showInfoMessage('', {
messageElement: $msg.clone().show(),
transient: false,
position: { my: 'bottom left', at: 'top center', offsetTop: -7 },
dismissable: false,
relativeToBody: true
});
},
function() {
StackExchange.helpers.removeMessages();
}
);
});
});
Jan 16 at 19:59


This question has been asked before and already has an answer. If those answers do not fully address your question, please ask a new question.


















  • $begingroup$
    An alternative to dividing this up into cases where the series ends in $2$ games, $3$ games and $4$ games, is to play all $4$ remaining games (some unnecessarily): this team need to win $2$, $3$ or $4$ of them to win the series
    $endgroup$
    – Henry
    Jan 16 at 18:23










  • $begingroup$
    I checked that thread out, but the solution given uses a tree or assuming all of the remaining 4 games get played out. I'm trying to use cases where not all 4 games necessarily get played out.
    $endgroup$
    – Panaso
    Jan 16 at 18:33














0












0








0





$begingroup$



This question already has an answer here:




  • If a sports team is down by 1-2 in a best of 7 series, what is their chance of winning (if both teams have a 1/2 chance of winning every game)?

    2 answers




Assuming that each team has an equal $frac{1}{2}$ probability of winning each game, and the probability of winning each game is independent.



I solved this using a tree diagram, and got the answer of $frac{11}{16}$, but I want to figure out a way of solving this without relying on a tree.



I tried diving this up to cases where the series ends in 2 games, 3 games and 4 games, but am having difficulty.



Any help would be greatly appreciated!










share|cite|improve this question











$endgroup$





This question already has an answer here:




  • If a sports team is down by 1-2 in a best of 7 series, what is their chance of winning (if both teams have a 1/2 chance of winning every game)?

    2 answers




Assuming that each team has an equal $frac{1}{2}$ probability of winning each game, and the probability of winning each game is independent.



I solved this using a tree diagram, and got the answer of $frac{11}{16}$, but I want to figure out a way of solving this without relying on a tree.



I tried diving this up to cases where the series ends in 2 games, 3 games and 4 games, but am having difficulty.



Any help would be greatly appreciated!





This question already has an answer here:




  • If a sports team is down by 1-2 in a best of 7 series, what is their chance of winning (if both teams have a 1/2 chance of winning every game)?

    2 answers








combinatorics statistics permutations






share|cite|improve this question















share|cite|improve this question













share|cite|improve this question




share|cite|improve this question








edited Jan 16 at 19:05









Key Flex

8,58571233




8,58571233










asked Jan 16 at 18:19









PanasoPanaso

143




143




marked as duplicate by David G. Stork, lulu, Key Flex, N. F. Taussig combinatorics
Users with the  combinatorics badge can single-handedly close combinatorics questions as duplicates and reopen them as needed.

StackExchange.ready(function() {
if (StackExchange.options.isMobile) return;

$('.dupe-hammer-message-hover:not(.hover-bound)').each(function() {
var $hover = $(this).addClass('hover-bound'),
$msg = $hover.siblings('.dupe-hammer-message');

$hover.hover(
function() {
$hover.showInfoMessage('', {
messageElement: $msg.clone().show(),
transient: false,
position: { my: 'bottom left', at: 'top center', offsetTop: -7 },
dismissable: false,
relativeToBody: true
});
},
function() {
StackExchange.helpers.removeMessages();
}
);
});
});
Jan 16 at 19:59


This question has been asked before and already has an answer. If those answers do not fully address your question, please ask a new question.









marked as duplicate by David G. Stork, lulu, Key Flex, N. F. Taussig combinatorics
Users with the  combinatorics badge can single-handedly close combinatorics questions as duplicates and reopen them as needed.

StackExchange.ready(function() {
if (StackExchange.options.isMobile) return;

$('.dupe-hammer-message-hover:not(.hover-bound)').each(function() {
var $hover = $(this).addClass('hover-bound'),
$msg = $hover.siblings('.dupe-hammer-message');

$hover.hover(
function() {
$hover.showInfoMessage('', {
messageElement: $msg.clone().show(),
transient: false,
position: { my: 'bottom left', at: 'top center', offsetTop: -7 },
dismissable: false,
relativeToBody: true
});
},
function() {
StackExchange.helpers.removeMessages();
}
);
});
});
Jan 16 at 19:59


This question has been asked before and already has an answer. If those answers do not fully address your question, please ask a new question.














  • $begingroup$
    An alternative to dividing this up into cases where the series ends in $2$ games, $3$ games and $4$ games, is to play all $4$ remaining games (some unnecessarily): this team need to win $2$, $3$ or $4$ of them to win the series
    $endgroup$
    – Henry
    Jan 16 at 18:23










  • $begingroup$
    I checked that thread out, but the solution given uses a tree or assuming all of the remaining 4 games get played out. I'm trying to use cases where not all 4 games necessarily get played out.
    $endgroup$
    – Panaso
    Jan 16 at 18:33


















  • $begingroup$
    An alternative to dividing this up into cases where the series ends in $2$ games, $3$ games and $4$ games, is to play all $4$ remaining games (some unnecessarily): this team need to win $2$, $3$ or $4$ of them to win the series
    $endgroup$
    – Henry
    Jan 16 at 18:23










  • $begingroup$
    I checked that thread out, but the solution given uses a tree or assuming all of the remaining 4 games get played out. I'm trying to use cases where not all 4 games necessarily get played out.
    $endgroup$
    – Panaso
    Jan 16 at 18:33
















$begingroup$
An alternative to dividing this up into cases where the series ends in $2$ games, $3$ games and $4$ games, is to play all $4$ remaining games (some unnecessarily): this team need to win $2$, $3$ or $4$ of them to win the series
$endgroup$
– Henry
Jan 16 at 18:23




$begingroup$
An alternative to dividing this up into cases where the series ends in $2$ games, $3$ games and $4$ games, is to play all $4$ remaining games (some unnecessarily): this team need to win $2$, $3$ or $4$ of them to win the series
$endgroup$
– Henry
Jan 16 at 18:23












$begingroup$
I checked that thread out, but the solution given uses a tree or assuming all of the remaining 4 games get played out. I'm trying to use cases where not all 4 games necessarily get played out.
$endgroup$
– Panaso
Jan 16 at 18:33




$begingroup$
I checked that thread out, but the solution given uses a tree or assuming all of the remaining 4 games get played out. I'm trying to use cases where not all 4 games necessarily get played out.
$endgroup$
– Panaso
Jan 16 at 18:33










2 Answers
2






active

oldest

votes


















0












$begingroup$

Note that the winning team necessarily wins the last game of the series. So there are three cases to consider:




  • The series ends in $5$ games. This occurs if the current leading team wins games $4$ and $5$, which occurs with probability $frac{1}{4}$

  • The series ends in $6$ games. Since they must win game $6$, they also must win exactly one of games $4,5$. The outcomes look like WLW, LWW. There are $2$, and each has probability $frac{1}{8}$ so this outcome occurs with probability $frac{1}{4}$

  • The series ends in $7$ games. Since they must win game $7$, they also must win exactly one of games $4,5,6$. The outcomes look like WLLW, LWLW, LLWW. There are $3$ outcomes and each has probability $frac{1}{16}$ so this situation occurs with probability $frac{3}{16}$


Therefore the answer is
$$frac{1}{4}+frac{1}{4}+frac{3}{16}=frac{11}{16}$$






share|cite|improve this answer









$endgroup$













  • $begingroup$
    Is there a way to calculate this with using combinations?
    $endgroup$
    – Panaso
    Jan 16 at 19:29










  • $begingroup$
    What do you mean by "combinations"? I simply wrote out all possible outcomes that satisfy the condition in your question.
    $endgroup$
    – pwerth
    Jan 16 at 19:31










  • $begingroup$
    I understand, but I'm required to put in combinations, such as 2C2.
    $endgroup$
    – Panaso
    Jan 16 at 19:49










  • $begingroup$
    I see. That wasn't included in your question so I took a different route. For future reference, if you need a question solved in a specific manner you should include that in your question.
    $endgroup$
    – pwerth
    Jan 16 at 19:52










  • $begingroup$
    Sorry about that, I'm not sure how to get the denominators using combinations. Could you give me a hint?
    $endgroup$
    – Panaso
    Jan 16 at 20:04



















0












$begingroup$

You can take a Markov chain approach. The possible states that describe the series are of the form $(m,n)$, where $m$ is the number of games won so far by team A and $n$ is the number of games won so far by team B. In your example, the series is in state $(2,1)$.



Let $P(m,n)$ denote the probability that team A will win the series, given it is in state $(m,n)$. Let's consider your specific example of $(2,1)$. Two outcomes are possible starting from this state (each with probability ${1 over 2}$. Team A can win game 4 and the series could go to state $(3,1)$ or Team B could win game 4 and the series could go the state $(2,2)$. You can express the probability relation as:



$$P(2,1) = {1 over 2}P(3,1) + {1 over 2}P(2,2)$$



By the same logic:
$$P(3,1) = {1 over 2}(1) + {1 over 2}P(3,2)$$
$$P(3,2) = {1 over 2}(1) + {1 over 2}P(3,3)$$
$$P(3,3) = {1 over 2}(1) + {1 over 2}(0)$$
$$P(2,2) = {1 over 2}P(3,2) + {1 over 2}P(2,3)$$
$$P(2,3) = {1 over 2}P(3,3) + {1 over 2}(0)$$



You have the necessary equations above to solve for $P(2,1)$ for the probability of winning from any state, for that matter.



Also, you can use matrix notation to express state transitions instead of the tedious equations I wrote above.






share|cite|improve this answer









$endgroup$




















    2 Answers
    2






    active

    oldest

    votes








    2 Answers
    2






    active

    oldest

    votes









    active

    oldest

    votes






    active

    oldest

    votes









    0












    $begingroup$

    Note that the winning team necessarily wins the last game of the series. So there are three cases to consider:




    • The series ends in $5$ games. This occurs if the current leading team wins games $4$ and $5$, which occurs with probability $frac{1}{4}$

    • The series ends in $6$ games. Since they must win game $6$, they also must win exactly one of games $4,5$. The outcomes look like WLW, LWW. There are $2$, and each has probability $frac{1}{8}$ so this outcome occurs with probability $frac{1}{4}$

    • The series ends in $7$ games. Since they must win game $7$, they also must win exactly one of games $4,5,6$. The outcomes look like WLLW, LWLW, LLWW. There are $3$ outcomes and each has probability $frac{1}{16}$ so this situation occurs with probability $frac{3}{16}$


    Therefore the answer is
    $$frac{1}{4}+frac{1}{4}+frac{3}{16}=frac{11}{16}$$






    share|cite|improve this answer









    $endgroup$













    • $begingroup$
      Is there a way to calculate this with using combinations?
      $endgroup$
      – Panaso
      Jan 16 at 19:29










    • $begingroup$
      What do you mean by "combinations"? I simply wrote out all possible outcomes that satisfy the condition in your question.
      $endgroup$
      – pwerth
      Jan 16 at 19:31










    • $begingroup$
      I understand, but I'm required to put in combinations, such as 2C2.
      $endgroup$
      – Panaso
      Jan 16 at 19:49










    • $begingroup$
      I see. That wasn't included in your question so I took a different route. For future reference, if you need a question solved in a specific manner you should include that in your question.
      $endgroup$
      – pwerth
      Jan 16 at 19:52










    • $begingroup$
      Sorry about that, I'm not sure how to get the denominators using combinations. Could you give me a hint?
      $endgroup$
      – Panaso
      Jan 16 at 20:04
















    0












    $begingroup$

    Note that the winning team necessarily wins the last game of the series. So there are three cases to consider:




    • The series ends in $5$ games. This occurs if the current leading team wins games $4$ and $5$, which occurs with probability $frac{1}{4}$

    • The series ends in $6$ games. Since they must win game $6$, they also must win exactly one of games $4,5$. The outcomes look like WLW, LWW. There are $2$, and each has probability $frac{1}{8}$ so this outcome occurs with probability $frac{1}{4}$

    • The series ends in $7$ games. Since they must win game $7$, they also must win exactly one of games $4,5,6$. The outcomes look like WLLW, LWLW, LLWW. There are $3$ outcomes and each has probability $frac{1}{16}$ so this situation occurs with probability $frac{3}{16}$


    Therefore the answer is
    $$frac{1}{4}+frac{1}{4}+frac{3}{16}=frac{11}{16}$$






    share|cite|improve this answer









    $endgroup$













    • $begingroup$
      Is there a way to calculate this with using combinations?
      $endgroup$
      – Panaso
      Jan 16 at 19:29










    • $begingroup$
      What do you mean by "combinations"? I simply wrote out all possible outcomes that satisfy the condition in your question.
      $endgroup$
      – pwerth
      Jan 16 at 19:31










    • $begingroup$
      I understand, but I'm required to put in combinations, such as 2C2.
      $endgroup$
      – Panaso
      Jan 16 at 19:49










    • $begingroup$
      I see. That wasn't included in your question so I took a different route. For future reference, if you need a question solved in a specific manner you should include that in your question.
      $endgroup$
      – pwerth
      Jan 16 at 19:52










    • $begingroup$
      Sorry about that, I'm not sure how to get the denominators using combinations. Could you give me a hint?
      $endgroup$
      – Panaso
      Jan 16 at 20:04














    0












    0








    0





    $begingroup$

    Note that the winning team necessarily wins the last game of the series. So there are three cases to consider:




    • The series ends in $5$ games. This occurs if the current leading team wins games $4$ and $5$, which occurs with probability $frac{1}{4}$

    • The series ends in $6$ games. Since they must win game $6$, they also must win exactly one of games $4,5$. The outcomes look like WLW, LWW. There are $2$, and each has probability $frac{1}{8}$ so this outcome occurs with probability $frac{1}{4}$

    • The series ends in $7$ games. Since they must win game $7$, they also must win exactly one of games $4,5,6$. The outcomes look like WLLW, LWLW, LLWW. There are $3$ outcomes and each has probability $frac{1}{16}$ so this situation occurs with probability $frac{3}{16}$


    Therefore the answer is
    $$frac{1}{4}+frac{1}{4}+frac{3}{16}=frac{11}{16}$$






    share|cite|improve this answer









    $endgroup$



    Note that the winning team necessarily wins the last game of the series. So there are three cases to consider:




    • The series ends in $5$ games. This occurs if the current leading team wins games $4$ and $5$, which occurs with probability $frac{1}{4}$

    • The series ends in $6$ games. Since they must win game $6$, they also must win exactly one of games $4,5$. The outcomes look like WLW, LWW. There are $2$, and each has probability $frac{1}{8}$ so this outcome occurs with probability $frac{1}{4}$

    • The series ends in $7$ games. Since they must win game $7$, they also must win exactly one of games $4,5,6$. The outcomes look like WLLW, LWLW, LLWW. There are $3$ outcomes and each has probability $frac{1}{16}$ so this situation occurs with probability $frac{3}{16}$


    Therefore the answer is
    $$frac{1}{4}+frac{1}{4}+frac{3}{16}=frac{11}{16}$$







    share|cite|improve this answer












    share|cite|improve this answer



    share|cite|improve this answer










    answered Jan 16 at 18:29









    pwerthpwerth

    3,340417




    3,340417












    • $begingroup$
      Is there a way to calculate this with using combinations?
      $endgroup$
      – Panaso
      Jan 16 at 19:29










    • $begingroup$
      What do you mean by "combinations"? I simply wrote out all possible outcomes that satisfy the condition in your question.
      $endgroup$
      – pwerth
      Jan 16 at 19:31










    • $begingroup$
      I understand, but I'm required to put in combinations, such as 2C2.
      $endgroup$
      – Panaso
      Jan 16 at 19:49










    • $begingroup$
      I see. That wasn't included in your question so I took a different route. For future reference, if you need a question solved in a specific manner you should include that in your question.
      $endgroup$
      – pwerth
      Jan 16 at 19:52










    • $begingroup$
      Sorry about that, I'm not sure how to get the denominators using combinations. Could you give me a hint?
      $endgroup$
      – Panaso
      Jan 16 at 20:04


















    • $begingroup$
      Is there a way to calculate this with using combinations?
      $endgroup$
      – Panaso
      Jan 16 at 19:29










    • $begingroup$
      What do you mean by "combinations"? I simply wrote out all possible outcomes that satisfy the condition in your question.
      $endgroup$
      – pwerth
      Jan 16 at 19:31










    • $begingroup$
      I understand, but I'm required to put in combinations, such as 2C2.
      $endgroup$
      – Panaso
      Jan 16 at 19:49










    • $begingroup$
      I see. That wasn't included in your question so I took a different route. For future reference, if you need a question solved in a specific manner you should include that in your question.
      $endgroup$
      – pwerth
      Jan 16 at 19:52










    • $begingroup$
      Sorry about that, I'm not sure how to get the denominators using combinations. Could you give me a hint?
      $endgroup$
      – Panaso
      Jan 16 at 20:04
















    $begingroup$
    Is there a way to calculate this with using combinations?
    $endgroup$
    – Panaso
    Jan 16 at 19:29




    $begingroup$
    Is there a way to calculate this with using combinations?
    $endgroup$
    – Panaso
    Jan 16 at 19:29












    $begingroup$
    What do you mean by "combinations"? I simply wrote out all possible outcomes that satisfy the condition in your question.
    $endgroup$
    – pwerth
    Jan 16 at 19:31




    $begingroup$
    What do you mean by "combinations"? I simply wrote out all possible outcomes that satisfy the condition in your question.
    $endgroup$
    – pwerth
    Jan 16 at 19:31












    $begingroup$
    I understand, but I'm required to put in combinations, such as 2C2.
    $endgroup$
    – Panaso
    Jan 16 at 19:49




    $begingroup$
    I understand, but I'm required to put in combinations, such as 2C2.
    $endgroup$
    – Panaso
    Jan 16 at 19:49












    $begingroup$
    I see. That wasn't included in your question so I took a different route. For future reference, if you need a question solved in a specific manner you should include that in your question.
    $endgroup$
    – pwerth
    Jan 16 at 19:52




    $begingroup$
    I see. That wasn't included in your question so I took a different route. For future reference, if you need a question solved in a specific manner you should include that in your question.
    $endgroup$
    – pwerth
    Jan 16 at 19:52












    $begingroup$
    Sorry about that, I'm not sure how to get the denominators using combinations. Could you give me a hint?
    $endgroup$
    – Panaso
    Jan 16 at 20:04




    $begingroup$
    Sorry about that, I'm not sure how to get the denominators using combinations. Could you give me a hint?
    $endgroup$
    – Panaso
    Jan 16 at 20:04











    0












    $begingroup$

    You can take a Markov chain approach. The possible states that describe the series are of the form $(m,n)$, where $m$ is the number of games won so far by team A and $n$ is the number of games won so far by team B. In your example, the series is in state $(2,1)$.



    Let $P(m,n)$ denote the probability that team A will win the series, given it is in state $(m,n)$. Let's consider your specific example of $(2,1)$. Two outcomes are possible starting from this state (each with probability ${1 over 2}$. Team A can win game 4 and the series could go to state $(3,1)$ or Team B could win game 4 and the series could go the state $(2,2)$. You can express the probability relation as:



    $$P(2,1) = {1 over 2}P(3,1) + {1 over 2}P(2,2)$$



    By the same logic:
    $$P(3,1) = {1 over 2}(1) + {1 over 2}P(3,2)$$
    $$P(3,2) = {1 over 2}(1) + {1 over 2}P(3,3)$$
    $$P(3,3) = {1 over 2}(1) + {1 over 2}(0)$$
    $$P(2,2) = {1 over 2}P(3,2) + {1 over 2}P(2,3)$$
    $$P(2,3) = {1 over 2}P(3,3) + {1 over 2}(0)$$



    You have the necessary equations above to solve for $P(2,1)$ for the probability of winning from any state, for that matter.



    Also, you can use matrix notation to express state transitions instead of the tedious equations I wrote above.






    share|cite|improve this answer









    $endgroup$


















      0












      $begingroup$

      You can take a Markov chain approach. The possible states that describe the series are of the form $(m,n)$, where $m$ is the number of games won so far by team A and $n$ is the number of games won so far by team B. In your example, the series is in state $(2,1)$.



      Let $P(m,n)$ denote the probability that team A will win the series, given it is in state $(m,n)$. Let's consider your specific example of $(2,1)$. Two outcomes are possible starting from this state (each with probability ${1 over 2}$. Team A can win game 4 and the series could go to state $(3,1)$ or Team B could win game 4 and the series could go the state $(2,2)$. You can express the probability relation as:



      $$P(2,1) = {1 over 2}P(3,1) + {1 over 2}P(2,2)$$



      By the same logic:
      $$P(3,1) = {1 over 2}(1) + {1 over 2}P(3,2)$$
      $$P(3,2) = {1 over 2}(1) + {1 over 2}P(3,3)$$
      $$P(3,3) = {1 over 2}(1) + {1 over 2}(0)$$
      $$P(2,2) = {1 over 2}P(3,2) + {1 over 2}P(2,3)$$
      $$P(2,3) = {1 over 2}P(3,3) + {1 over 2}(0)$$



      You have the necessary equations above to solve for $P(2,1)$ for the probability of winning from any state, for that matter.



      Also, you can use matrix notation to express state transitions instead of the tedious equations I wrote above.






      share|cite|improve this answer









      $endgroup$
















        0












        0








        0





        $begingroup$

        You can take a Markov chain approach. The possible states that describe the series are of the form $(m,n)$, where $m$ is the number of games won so far by team A and $n$ is the number of games won so far by team B. In your example, the series is in state $(2,1)$.



        Let $P(m,n)$ denote the probability that team A will win the series, given it is in state $(m,n)$. Let's consider your specific example of $(2,1)$. Two outcomes are possible starting from this state (each with probability ${1 over 2}$. Team A can win game 4 and the series could go to state $(3,1)$ or Team B could win game 4 and the series could go the state $(2,2)$. You can express the probability relation as:



        $$P(2,1) = {1 over 2}P(3,1) + {1 over 2}P(2,2)$$



        By the same logic:
        $$P(3,1) = {1 over 2}(1) + {1 over 2}P(3,2)$$
        $$P(3,2) = {1 over 2}(1) + {1 over 2}P(3,3)$$
        $$P(3,3) = {1 over 2}(1) + {1 over 2}(0)$$
        $$P(2,2) = {1 over 2}P(3,2) + {1 over 2}P(2,3)$$
        $$P(2,3) = {1 over 2}P(3,3) + {1 over 2}(0)$$



        You have the necessary equations above to solve for $P(2,1)$ for the probability of winning from any state, for that matter.



        Also, you can use matrix notation to express state transitions instead of the tedious equations I wrote above.






        share|cite|improve this answer









        $endgroup$



        You can take a Markov chain approach. The possible states that describe the series are of the form $(m,n)$, where $m$ is the number of games won so far by team A and $n$ is the number of games won so far by team B. In your example, the series is in state $(2,1)$.



        Let $P(m,n)$ denote the probability that team A will win the series, given it is in state $(m,n)$. Let's consider your specific example of $(2,1)$. Two outcomes are possible starting from this state (each with probability ${1 over 2}$. Team A can win game 4 and the series could go to state $(3,1)$ or Team B could win game 4 and the series could go the state $(2,2)$. You can express the probability relation as:



        $$P(2,1) = {1 over 2}P(3,1) + {1 over 2}P(2,2)$$



        By the same logic:
        $$P(3,1) = {1 over 2}(1) + {1 over 2}P(3,2)$$
        $$P(3,2) = {1 over 2}(1) + {1 over 2}P(3,3)$$
        $$P(3,3) = {1 over 2}(1) + {1 over 2}(0)$$
        $$P(2,2) = {1 over 2}P(3,2) + {1 over 2}P(2,3)$$
        $$P(2,3) = {1 over 2}P(3,3) + {1 over 2}(0)$$



        You have the necessary equations above to solve for $P(2,1)$ for the probability of winning from any state, for that matter.



        Also, you can use matrix notation to express state transitions instead of the tedious equations I wrote above.







        share|cite|improve this answer












        share|cite|improve this answer



        share|cite|improve this answer










        answered Jan 16 at 18:35









        Aditya DuaAditya Dua

        1,15418




        1,15418















            Popular posts from this blog

            Human spaceflight

            Can not write log (Is /dev/pts mounted?) - openpty in Ubuntu-on-Windows?

            File:DeusFollowingSea.jpg